Fiche de mathématiques
> >

Mathématiques Bac ES-L Pondichéry 2018

Obligatoire et Spécialité

Partager :

sujet mathématiques bac ES-L pondichéry 2018 obligatoire et spécialité : image 2

sujet mathématiques bac ES-L pondichéry 2018 obligatoire et spécialité : image 16

sujet mathématiques bac ES-L pondichéry 2018 obligatoire et spécialité : image 13

sujet mathématiques bac ES-L pondichéry 2018 obligatoire et spécialité : image 12

sujet mathématiques bac ES-L pondichéry 2018 obligatoire et spécialité : image 17

sujet mathématiques bac ES-L pondichéry 2018 obligatoire et spécialité : image 1

sujet mathématiques bac ES-L pondichéry 2018 obligatoire et spécialité : image 7

sujet mathématiques bac ES-L pondichéry 2018 obligatoire et spécialité : image 3

sujet mathématiques bac ES-L pondichéry 2018 obligatoire et spécialité : image 6

sujet mathématiques bac ES-L pondichéry 2018 obligatoire et spécialité : image 8

sujet mathématiques bac ES-L pondichéry 2018 obligatoire et spécialité : image 10

sujet mathématiques bac ES-L pondichéry 2018 obligatoire et spécialité : image 15

sujet mathématiques bac ES-L pondichéry 2018 obligatoire et spécialité : image 4

sujet mathématiques bac ES-L pondichéry 2018 obligatoire et spécialité : image 9

sujet mathématiques bac ES-L pondichéry 2018 obligatoire et spécialité : image 5

sujet mathématiques bac ES-L pondichéry 2018 obligatoire et spécialité : image 14

sujet mathématiques bac ES-L pondichéry 2018 obligatoire et spécialité : image 11






Bac ES-L Pondichéry 2018 - Obligatoire et Spécialité

Partager :


5 points

exercice 1

1. f'(x)=\dfrac{-5\ln x}{x^2}

Puisque x ² > 0, le signe de f'  sera le signe de -5ln x .

\left\lbrace\begin{matrix}\ln x\le0\ \ \text{si }\ 0<x\le1\\ \ln x\ge0\ \ \text{si }\ x\ge1\end{matrix}\right.\Longleftrightarrow\left\lbrace\begin{matrix}-5\ln x\ge0\ \ \text{si }\ 0<x\le1\\ -5\ln x\le0\ \ \text{si }\ x\ge1\end{matrix}\right.\\\\\phantom{\left\lbrace\begin{matrix}\ln x\le0\ \ \text{si }\ 0<x\le1\\ \ln x\ge0\ \ \text{si }\ x\ge1\end{matrix}\right.}\Longrightarrow-5\ln x\le0\ \ \text{si }\ x\in[1;+\infty[\\\phantom{\left\lbrace\begin{matrix}\ln x\le0\ \ \text{si }\ 0<x\le1\\ \ln x\ge0\ \ \text{si }\ x\ge1\end{matrix}\right.}\Longrightarrow-5\ln x\le0\ \ \text{si }\ x\in[1;5]\\\phantom{\left\lbrace\begin{matrix}\ln x\le0\ \ \text{si }\ 0<x\le1\\ \ln x\ge0\ \ \text{si }\ x\ge1\end{matrix}\right.}\Longrightarrow\boxed{f'(x)\le0\ \ \text{si }\ x\in[1;5]}

Par conséquent la réponse exacte est la réponse  \text{{\red{(b)}}.}

2.   L'abscisse du point B  est égale à e.
Le coefficient directeur de la tangente à la courbe  \mathscr{C}  au point B  est égal à f' (e).

f'(x)=\dfrac{-5\ln x}{x^2}\Longrightarrow f'(\text{e})=\dfrac{-5\ln \text{e}}{\text{e}^2}\\\\\phantom{f'(x)=\dfrac{-5\ln x}{x^2}}\Longrightarrow f'(\text{e})=\dfrac{-5\times1}{\text{e}^2}\\\\\phantom{f'(x)=\dfrac{-5\ln x}{x^2}}\Longrightarrow\boxed{f'(\text{e})=\dfrac{-5}{\text{e}^2}}

Par conséquent la réponse exacte est la réponse  \text{{\red{(a)}}.}

3.   La croissance de f'  se déduit de l'étude du signe de f" (x ) où f
Puisque x  appartient [0,5 ; 5], nous savons que x 3 > 0.
D'où le signe de f" (x ) sera le signe de 10 ln x  - 5.

\begin{array}l10\ln x-5=0\Longleftrightarrow10\ln x=5\\\phantom{10\ln x-5=0}\Longleftrightarrow\ln x=0,5\\\phantom{10\ln x-5=0}\Longleftrightarrow x=\text{e}^{0,5} \end{array}\begin{array}l\ |\ \\\ |\ \\\ |\ \\\ |\  \end{array}\begin{array}l10\ln x-5\le0\Longleftrightarrow10\ln x\le5\\\phantom{10\ln x-5\le0}\Longleftrightarrow\ln x\le0,5\\\phantom{10\ln x-5=0}\Longleftrightarrow x\le\text{e}^{0,5} \end{array}\begin{array}l\ |\ \\\ |\ \\\ |\ \\\ |\  \end{array}\begin{array}l10\ln x-5\ge0\Longleftrightarrow10\ln x\ge5\\\phantom{10\ln x-5=0}\Longleftrightarrow\ln x\ge0,5\\\phantom{10\ln x-5=0}\Longleftrightarrow x\ge\text{e}^{0,5} \end{array}

Tableau de signes de f" (x ) sur l'intervalle [0,5 ; 5] :

\begin{array}{|c|ccccc|}\hline x&0,5&&\text{e}^{0,5}\approx1,6&&5 \\\hline f''(x)&&-&0&+&\\\hline f'(x)&&\searrow&&\nearrow&\\\hline \end{array}

Donc la fonction f'  est croissante sur l'intervalle [2 ; 5].
Par conséquent la réponse exacte est la réponse  \text{{\red{(c)}}.}

4.   La fonction f  est deux fois dérivable sur l'intervalle [0,5 ; 5].
La courbe  \mathscr{C}  admet le point A  comme seul point d'inflexion.
Donc la dérivée seconde s'annule en l'abscisse de ce point A .

Nous connaîtrons la valeur de cette abscisse en résolvant l'équation f" (x ) = 0.
En utilisant les résultats de la question 3, nous obtenons :

f''(x)=0\Longleftrightarrow\dfrac{10\ln x-5}{x^3}=0\\\\\phantom{f''(x)=0}\Longleftrightarrow10\ln x-5=0\\\\\phantom{f''(x)=0}\Longleftrightarrow\boxed{x=e^{0,5}}

Par conséquent la réponse exacte est la réponse  \text{{\red{(c)}}.}

5.   Le graphique ci-dessous montre clairement que l'aire  \mathscr{A}  en pointillé rouge est comprise entre l'aire du polygone DEFHIJ  bleuté et le rectangle DKLJ  de couleur verte.

sujet mathématiques bac ES-L pondichéry 2018 obligatoire et spécialité : image 18


Or le polygone DEFHIJ  est composé de 20 pavés de 0,5 u. a.
Donc l'aire de ce polygone DEFHIJ  est égale à 10 u. a.
Le rectangle DKLJ est de dimensions 3 multiplie  5.
Donc l'aire de ce rectangle DKLJ  est égale à 15 u. a.

D'où  10<\mathscr{A}<15.
Par conséquent la réponse exacte est la réponse  \text{{\red{(b)}}.}

5 points

exercice 2

Partie A

1. a)   80 % des clients règlent des sommes inférieures ou égales à 30 euros.

Donc  \boxed{P(V)=0,8}

Parmi les clients réglant des sommes inférieures ou égales à 30 euros, 40 % paient avec une carte bancaire en mode sans contact.

Donc \boxed{P_V(S)=0,4}

b)   Arbre pondéré représentant la situation :

sujet mathématiques bac ES-L pondichéry 2018 obligatoire et spécialité : image 20


2. a)   La probabilité que pour son achat, le client ait réglé un montant inférieur ou égal à 30 euros et qu'il ait utilisé sa carte bancaire en mode sans contact correspond à  P(V\cap S).

P(V\cap S)=P(V)\times P_V(S)\\\phantom{P(V\cap S)}=0,8\times0,4\\\phantom{P(V\cap S)}=0,32\\\\\Longrightarrow\boxed{P(V\cap S)=0,32}

b)   Notons par M  l'événement "pour son achat, le client a réglé avec sa carte bancaire en utilisant l'un des deux modes".
M  est l'événement contraire de l'événement E.
Donc P(M)=1-P(E)
Or en utilisant la formule des probabilités totales, nous avons :

P(E)=P(V\cap E)+P(\overline{V}\cap E)\\\phantom{P(E)}=P(V)\times P_V(E)+P(\overline{V})\times P_{\overline{V}}(E)\\\phantom{P(E)}=0,8\times0,4+0,2\times0,3\\\phantom{P(E)}=0,38\\\\\Longrightarrow P(E)=0,38

\text{D'où }\ \ P(M)=1-0,38\\\phantom{\text{D'où }\ \ }\boxed{P(M)=0,62}

Par conséquent, la probabilité de l'événement "pour son achat, le client a réglé avec sa carte bancaire en utilisant l'un des deux modes" est égale à 0,62.

Partie B

X  suit la loi normale de moyenne mu = 27,5 et d'écart type sigma = 3.

{\red{1.}}\ \ P(X<30)=P(X\le 27,5)+P(27,5<X<30)\\\phantom{{\red{1.}}\ \ P(X<30)}=0,5+P(27,5<X<30)\\\phantom{{\red{1.}}\ \ P(X<30)}\approx0,5+0.298\\\phantom{{\red{1.}}\ \ P(X<30)}\approx0,798\\\\\Longrightarrow\boxed{P(X<30)\approx0,798}

Donc la probabilité que ce client ait dépensé moins de 30 euros est environ égale à 0,80 (arrondie à 0,01 près).

2.   Par la calculatrice, nous obtenons  P(24,5\le X\le30,5)\approx0,68268949
D'où la probabilité que ce client ait dépensé entre 24,5 euros et 30,5 euros est environ égale à 0,68 (arrondie à 0,01 près).

Nous pouvions trouver ce résultat par la propriété suivante de la loi normale :

P(\mu-\sigma\le X\le\mu+\sigma)\approx0,68.

Nous obtenons alors :

P(24,5\le X\le30,5)=P(27,5-3\le X\le27,5+3)\\\phantom{P(81,6\le X\le82,4)}=P(\mu-\sigma\le X\le\mu+\sigma)\\\phantom{P(81,6\le X\le82,4)}\approx0,68

Partie C

Déterminons l'intervalle de confiance  I_{200}  au seuil de 95% de la proportion p  de clients qui trouvent que ce dispositif sans contact est pratique.

La fréquence des clients de l'échantillon trouvant que le dispositif sans contact est pratique est donnée par

f=\dfrac{175}{200}=0,875.

L'intervalle de confiance  I_{200}  au seuil de 95% est   I_{200}=\left[0,875-\dfrac{1}{\sqrt{200}};0,875+\dfrac{1}{\sqrt{200}}\right]

\Longrightarrow\boxed{I_{200}=[0,80\ ;\ 0,95]}

Les conditions d'utilisation de l'intervalle de confiance sont remplies.
En effet,

\left\lbrace\begin{array}l n=200\Longrightarrow {\red{n\ge30}} \\p\in[0,80;0,95]\Longrightarrow0,80\le p\le0,95\\\phantom{p\in[0,80;0,95]}\Longrightarrow200\times0,80\le np\le200\times0,95\\\phantom{p\in[0,80;0,95]}\Longrightarrow160\le np\le190\\\phantom{p\in[0,80;0,95]}\Longrightarrow{\red{ np\ge5}} \\p\in[0,80;0,95]\Longrightarrow0,80\le p\le0,95\\\phantom{p\in[0,80;0,95]}\Longrightarrow-0,95\le-p\le-0,80\\\phantom{p\in[0,80;0,95]}\Longrightarrow1-0,95\le1-p\le1-0,80\\\phantom{p\in[0,80;0,95]}\Longrightarrow0,05\le1-p\le0,20\\\phantom{p\in[0,80;0,95]}\Longrightarrow200\times0,05\le n(1-p)\le200\times0,20\\\phantom{p\in[0,80;0,95]}\Longrightarrow10\le n(1-p)\le40\\\phantom{p\in[0,80;0,95]}\Longrightarrow{\red{ n(1-p)\ge5}}  \end{array}

5 points

exercice 3

\left\lbrace\begin{array}l u_0=65\\u_{n+1}=0,8u_n+18 \end{array}

{\red{1.}}\ \ u_1=0,8\times u_0+18\\\phantom{{\red{1.}}\ \ u_1}=0,8\times65+18\\\phantom{{\red{1.}}\ \ u_1}=70\\\\\Longrightarrow\boxed{u_1=70} \\\\u_2=0,8\times u_1+18\\\phantom{u_1}=0,8\times70+18\\\phantom{u_1}=74\\\\\Longrightarrow\boxed{u_2=74}

2.   v_n=u_n-90

a.   Montrons que la suite (vn ) est une suite géométrique.

v_{n+1}=u_{n+1}-90\\\phantom{v_{n+1}}=(0,8\times u_n+18)-90\\\phantom{v_{n+1}}=0,8\times u_n-72\\\phantom{v_{n+1}}=0,8\times u_n-0,8\times90\\\phantom{v_{n+1}}=0,8\times (u_n-90)\\\phantom{v_{n+1}}=0,8\times v_n\\\\\Longrightarrow\boxed{v_{n+1}=0,8\times v_n}

Par conséquent la suite (vn ) est une suite géométrique de raison 0,8 et dont le premier terme est v 0=u 0-90 = 65-90 = -25.

b.   Le terme général de la suite (vn ) est  v_n=v_0\times0,8^n=-25\times0,8^n

\text{D'où  }\ \ v_n=u_n-90\Longrightarrow u_n=v_n+90\\\phantom{\text{D'\ \ où  }v_n=u_n-90}\Longrightarrow u_n=-25\times0,8^n+90\\\\\Longrightarrow\boxed{u_n=90-25\times0,8^n}

3. a)   Algorithme complété :

\begin{array}{|c|}\hline \text{ligne 1}\\\text{ligne 2}\\\text{ligne 3}\\\text{ligne 4}\\\text{ligne 5}\\\text{ligne 6}\\\hline \end{array}\begin{array}{|c|}\hline u\longleftarrow65\\n\longleftarrow0\\\ \ \ \ \ \ \ \ \ \ \text{Tant que }{\red{u<85}}\\\ \ \ \ \ \ \ \ \ \ \ \ \ \ \ \ \ \ n\longleftarrow n+1\\\ \ \ \ \ \ \ \ \ \ \ \ \ \ \ \ \ \ \ \ \ \ \ \ \ \ \ \ u\longleftarrow 0,8\times u+18\\\ \ \ \ \ \text{Fin Tant que}\\\hline \end{array}

b)   Les diverses valeurs prises par u (arrondies au centième) durant l'exécution de l'algorithme sont reprises dans le tableau suivant :

\begin{array}{|c|c|c|c|c|c|c|c|c|c|c|c|c|c|}\hline n&0&1&2&3&4&5&6&7&8&9\\\hline u&65&70&74&77,2&79,76&81,81&83,45&84,76&{\red{85,81}}&{\red{...}}\\\hline \end{array}

La boucle "Tant que " s'arrête dès que u  n'est plus strictement inférieur à 85.
Donc la valeur de n  fournie à la fin de l'exécution de l'algorithme est n  = 8.

c.   Résolvons l'inéquation un supegal  85.

90-25\times0,8^n\ge85\Longleftrightarrow-25\times0,8^n\ge85-90\\\phantom{90-25\times0,8^n\ge85}\Longleftrightarrow-25\times0,8^n\ge-5\\\\\phantom{90-25\times0,8^n\ge85}\Longleftrightarrow0,8^n\le\dfrac{-5}{-25}\\\\\phantom{90-25\times0,8^n\ge85}\Longleftrightarrow0,8^n\le0,2\\\phantom{90-25\times0,8^n\ge85}\Longleftrightarrow\ln(0,8^n)\le\ln 0,2\\\phantom{90-25\times0,8^n\ge85}\Longleftrightarrow n\ln0,8\le\ln0,2\\\\\phantom{90-25\times0,8^n\ge85}\Longleftrightarrow n\ {\red{\ge}}\ \dfrac{\ln0,2}{\ln0,8}\ \ \ \text{(car  }\ln0,8<0)\\\\\text{Or }\ \ \dfrac{\ln0,2}{\ln0,8}\approx7,21

D'où le plus petit entier naturel vérifiant l'inéquation un supegal  85 est n = 8.
Nous retrouvons ainsi le résultat obtenu à la question précédente.

4. a)   En juillet 2017, 65 particuliers ont souscrit à cet abonnement.
Donc u0 = 65.

Une diminution de 20 % correspond à un coefficient multiplicateur égal à 1 - 0,2 = 0,8.
D'où puisque d'un mois à l'autre, environ 20 % des abonnements sont résiliés, le nombre d'abonnés un  sera multiplié par 0,8, soit 0,8un.

Ensuite chaque mois, 18 particuliers supplémentaires souscrivent à l'abonnement.
Nous ajouterons alors 18 à 0,8un. Par conséquent  u_{n+1}=0,8u_n+18.

La suite (un ) modélise parfaitement le nombre d'abonnés le n -ième mois qui suit le mois de juillet 2017.

b)   Nous avons montré dans la question 3c que le plus petit entier naturel vérifiant l'inéquation un supegal  85 est n=8.

Cela signifie que le 8ième  mois qui suit le mois de juillet 2017, le nombre d'abonnés sera supérieur à 85.
Dans ce cas, la recette mensuelle sera supérieure à 85 multiplie 52 euros, soit 4420 euros.

Le 8ième  mois qui suit le mois de juillet 2017 correspond au mois de mars 2018.
Donc à partir du mois de mars 2018, la recette mensuelle dépassera 4420 euros.

Par conséquent, la recette mensuelle de la société Biocagette va dépasser 4420 euros durant l'année 2018.

c)   Calculons  \lim\limits_{n\to+\infty}u_n

\lim\limits_{n\to+\infty}0,8^n=0\ \ \ \text{car }0<0,8<1\\\\\Longrightarrow\lim\limits_{n\to+\infty}(90-25\times0,8^n)=90-25\times\lim\limits_{n\to+\infty}0,8^n\\\phantom{\Longrightarrow\lim\limits_{n\to+\infty}(-25\times0,8^n)}=90-25\times0\\\phantom{\Longrightarrow\lim\limits_{n\to+\infty}(-25\times0,8^n)}=90\\\\\Longrightarrow\boxed{\lim\limits_{n\to+\infty}u_n=90}

Donc à long terme, le nombre mensuel d'abonnés tendra vers 90.

Dès lors, la recette mensuelle tendra vers 90 multiplie 52 euros, soit 4680 euros.

5 points

exercice 3

Partie A

Valeurs obtenues en utilisant l'algorithme de Dijkstra :

          \begin{array}{|c|c|c|c|c|c|c|c|c|c}\hline&A\ \ &\ \ B\ \ &\ \ C\ \ &\ \ D\ \ &\ \ E\ \ &\ \ F\ \ &\ \ H\ \ &\ \ G\ \ \\\hline A &\ \ \red{0_A}\ \ &\infty&\infty&\infty&\infty&\infty&\infty&\infty\\\hline E&&47_A&56_A&\infty&\red{23_A}&30_A&\infty&\infty\\\hline F&&43_E&56_A&65_E&&\red{30_A}&63_E&\infty\\\hline B&&\red{43_E}&56_A&65_E&&&58_F&\infty\\\hline C&& &\red{53_B}&65_E&&&58_F&\infty\\\hline H&&&&65_E&&&\red{58_F}&\infty\\\hline D&&&&\red{65_E}&&&&81_H\\\hline G&&&&&&&&\red{80_D}\\ \hline \end{array}

D'où le chemin le plus court qui permet à Louis de relier son domicile à son travail est A - E - D - G.
Louis parcourra alors une distance de 80 km.

Partie B

1. a)   Puisque le 1er janvier 2018, Louis décide d'utiliser le covoiturage, l'état probabiliste initial est P0 = (1   0).

b)   Les données de l'énoncé nous permettent de traduire la situation par le graphe probabiliste suivant :

sujet mathématiques bac ES-L pondichéry 2018 obligatoire et spécialité : image 19


2.   La matrice de transition du graphe probabiliste est \boxed{M=\begin{pmatrix}0,22 & 0,78\\0,53 & 0,47\end{pmatrix}}.

3.   Calcul de l'état probabiliste P 2.

P_2=P_0\times M^2\ \ \text{avec }P_0=(1\ \ \ \ \ 0)\\\\\text{Or  }M=\begin{pmatrix}0,22&0,78\\0,53&0,47\end{pmatrix}\Longrightarrow M^2=\begin{pmatrix}0,4618&0,5382\\0,3657&0,6343\end{pmatrix}\\\\\text{D'où  }P_2=(1\ \ \ \ \ 0)\times\begin{pmatrix}0,4618&0,5382\\0,3657&0,6343\end{pmatrix}\\\\\phantom{\text{D'où  }P_3}=(1\times0,4618+0\times0,3657\ \ \ \ 1\times0,5382+0\times0,6343)\\\\\phantom{\text{D'où  }P_3}=(0,4618\ \ \ \ 0,5382)\\\\\Longrightarrow\boxed{P_2=(0,4618\ \ \ 0,5382)}

Donc après 2 jours, soit le 3 janvier, Louis utilisera le covoiturage avec une probabilité de 46,18 % et il utilisera les transports en commun avec une probabilité de 53,82 %.

4) a)   La matrice M  de transition ne comporte pas de 0.
L'état probabiliste Pn  à l'étape n  converge vers un état P  indépendant de l'état initial P0 .

Cet état P  est l'état probabiliste stable du système et vérifie la relation PmultiplieM = P.

Soit   P=\begin{pmatrix}x & y\end{pmatrix}\ \ \ \text{avec }x+y=1

Alors

 P\times M=P

\Longleftrightarrow\begin{pmatrix}x & y\end{pmatrix}\times\begin{pmatrix}0,22&0,78 \\ 0,53 & 0,47\end{pmatrix}=\begin{pmatrix}x & y\end{pmatrix}\ \ \ \text{avec }x+y=1

\Longleftrightarrow\begin{pmatrix}0,22x+0,53y & 0,78x+0,47y\end{pmatrix}=\begin{pmatrix}x & y\end{pmatrix}\ \ \ \text{avec }x+y=1

\Longleftrightarrow\left\lbrace\begin{array}l 0,22x+0,53y=x\\0,78x+0,47y=y\\x+y=1 \end{array}

\Longleftrightarrow\left\lbrace\begin{array}l 0,22x-x+0,53y=0\\0,78x+0,47y-y=0\\x+y=1 \end{array}

\Longleftrightarrow\left\lbrace\begin{array}l -0,78x+0,53y=0\\0,78x-0,53y=0\\x+y=1 \end{array}

\Longleftrightarrow\left\lbrace\begin{array}l0,78x-0,53y=0\\x+y=1 \end{array}

\Longleftrightarrow\left\lbrace\begin{array}l0,78x-0,53y=0\\y=1-x \end{array}\Longleftrightarrow\left\lbrace\begin{array}l0,78x-0,53(1-x)=0\\y=1-x \end{array}

\Longleftrightarrow\left\lbrace\begin{array}l0,78x-0,53+0,53x=0\\y=1-x \end{array}\Longleftrightarrow\left\lbrace\begin{array}l1,31x-0,53=0\\y=1-x \end{array}\Longleftrightarrow\left\lbrace\begin{array}l1,31x=0,53\\y=1-x \end{array}

 \Longleftrightarrow\left\lbrace\begin{array}lx=\dfrac{0,53}{1,31}\\\\y=1-x \end{array}\Longleftrightarrow\left\lbrace\begin{array}lx=\dfrac{53}{131}\\\\y=1-\dfrac{53}{131} \end{array}\Longleftrightarrow\boxed{\left\lbrace\begin{array}lx=\dfrac{53}{131}\\\\y=\dfrac{78}{131} \end{array}}

D'où l'état probabiliste stable est  \boxed{P=\begin{pmatrix}\dfrac{53}{131} & \dfrac{78}{131}\end{pmatrix}\approx\begin{pmatrix}0,4 & 0,6\end{pmatrix}}

b)   Nous déduisons de la question précédente qu'à long terme, Louis utilisera le covoiturage avec une probabilité de 40 % et utilisera les transports en communs avec une probabilité de 60 %.
Donc à long terme, Louis n'utilisera pas aussi souvent le covoiturage que les transports en commun.

5 points

exercice 4

f(x)=(3,6x+2,4)e^{-0,6x}-1,4

Partie A

1.   Calcul de f' (x ).

f'(x)=[(3,6x+2,4)e^{-0,6x}]'-0\\\\\phantom{f'(x)}=(3,6x+2,4)'\times e^{-0,6x}+(3,6x+2,4)\times (e^{-0,6x})'\\\\\phantom{f'(x)}=3,6\times e^{-0,6x}+(3,6x+2,4)\times (-0,6\times e^{-0,6x})\\\\\phantom{f'(x)}=3,6\times e^{-0,6x}-3,6x\times 0,6\times e^{-0,6x}-2,4\times 0,6\times e^{-0,6x}\\\\\phantom{f'(x)}=3,6\times e^{-0,6x}-2,16x\times e^{-0,6x}-1,44\times e^{-0,6x}\\\\\phantom{f'(x)}=-2,16x\times e^{-0,6x}+2,16\times e^{-0,6x}\\\\\phantom{f'(x)}=(-2,16x+2,16)\times e^{-0,6x}\\\\\Longrightarrow\boxed{f'(x)=(-2,16x+2,16)e^{-0,6x}}

2. a)   Signe de f' (x ) sur [0 ; 4].

Puisque l'exponentielle est strictement positive, le signe de f' (x ) sera le signe de (-2,16x  + 2,16).

\begin{array}l{\red{-2,16x+2,16\le0}}\Longleftrightarrow 2,16x\ge2,16\\\\\phantom{-2,16x+2,16=0}\Longleftrightarrow x\ge\dfrac{2,16}{2,16}\\\\\phantom{-2,16x+2,16=0}\Longleftrightarrow {\red{x\ge1}}\end{array}\begin{array}l\ |\ \\\ |\ \\\ |\ \\\ |\ \\\ |\ \\\ | \end{array}\begin{array}l{\red{-2,16x+2,16\ge0}}\Longleftrightarrow 2,16x\le2,16\\\\\phantom{-2,16x+2,16=0}\Longleftrightarrow x\le\dfrac{2,16}{2,16}\\\\\phantom{-2,16x+2,16=0}\Longleftrightarrow {\red{x\le1}}\end{array}

Par conséquent f' (x ) supegal 0 sur l'intervalle [0 ; 1]
                                   f' (x ) infegal 0 sur l'intervalle [1 ; 4].

b)   Tableau de variation de f  sur l'intervalle [0 ; 4].

        \begin{array}{|c|ccccc|}\hline x&0&&1&&4\\\hline f'(x)&&+&0&-&\\\hline &&&\approx1,893&& \\ f(x)&&\nearrow&&\searrow &\\ &1&&&&\approx0,124 \\ \hline \end{array}

3.  Nous savons qu'une primitive F  de la fonction f  sur l'intervalle [0 ; 4] est définie par

F(x)=(-6x-14)e^{-0,6x}-1,4x.

\text{D'où }\ \ \int\limits_0^4f(x)\,dx=[F(x)]\mimits_0^4\\\phantom{\text{D'où }\ \ \int\limits_0^4f(x)\,dx}=F(4)-F(0)\\\phantom{\text{D'où }\ \ \int\limits_0^4f(x)\,dx}=[(-6\times4-14)e^{-0,6\times4}-1,4\times4]-[(-6\times0-14)e^{-0,6\times0}-1,4\times0]\\\phantom{\text{D'où }\ \ \int\limits_0^4f(x)\,dx}=(-38e^{-2,4}-5,6]-[(-14)e^0-0]\\\phantom{\text{D'où }\ \ \int\limits_0^4f(x)\,dx}=-38e^{-2,4}-5,6+14\\\phantom{\text{D'où }\ \ \int\limits_0^4f(x)\,dx}=8,4-38e^{-2,4}\\\\\Longrightarrow\boxed{\int\limits_0^4f(x)\,dx=8,4-38e^{-2,4}\approx4,95}

Partie B

g(x)=4x^2-4x+1

1.  Une primitive G  de la fonction g  sur l'intervalle [0 ; 4] est définie par

G(x)=\dfrac{4}{3}x^3-2x^2+x.

\text{D'où }\ \ \int\limits_0^{0,5}g(x)\,dx=[G(x)]\mimits_0^{0,5}\\\phantom{\text{D'où }\ \ \int\limits_0^4f(x)\,dx}=G({0,5})-G(0)\\\phantom{\text{D'où }\ \ \int\limits_0^4f(x)\,dx}=[\dfrac{4}{3}\times {0,5}^3-2\times{0,5}^2+{0,5}]-[\dfrac{4}{3}\times 0^3-2\times0^2+0]\\\phantom{\text{D'où }\ \ \int\limits_0^4f(x)\,dx}=\dfrac{4}{3}\times (\dfrac{1}{2})^3-2\times(\dfrac{1}{2})^2+\dfrac{1}{2}\\\phantom{\text{D'où }\ \ \int\limits_0^4f(x)\,dx}=\dfrac{4}{3}\times \dfrac{1}{8}-2\times\dfrac{1}{4}+\dfrac{1}{2}\\\phantom{\text{D'où }\ \ \int\limits_0^4f(x)\,dx}=\dfrac{1}{6}\\\\\Longrightarrow\boxed{\int\limits_0^{0,5}g(x)\,dx=\dfrac{1}{6}}

2.   Les fonctions f  et g  sont positives sur leurs ensembles de définition.

L'aire du domaine délimité par les courbes  \mathscr{C}_f , \mathscr{C}_g , la droite d'équation x = 4 et l'axe des abscisses se détermine par  \int\limits_0^{4}f(x)\,dx-\int\limits_0^{0,5}g(x)\,dx .

\int\limits_0^{4}f(x)\,dx-\int\limits_0^{0,5}g(x)\,dx\approx4,95-\dfrac{1}{6}\\\\\Longrightarrow\int\limits_0^{4}f(x)\,dx-\int\limits_0^{0,5}g(x)\,dx\approx4,783

En utilisant la symétrie des courbes  \mathscr{C}_f et \mathscr{C}_g par rapport à l'axe des abscisses, nous déterminerons l'aire cherchée en multipliant le résultat précédent par 2.

2\times4,783=9,566.

Par conséquent une valeur approchée de l'aire du domaine plan délimité par les courbes  \mathscr{C}_f , \mathscr{C}_g , leurs courbes symétriques (en pointillés) ainsi que la droite d'équation  x  = 4 est environ égale à 9,57 u.a. (arrondi à 0,01 près).
Publié le
ceci n'est qu'un extrait
Pour visualiser la totalité des cours vous devez vous inscrire / connecter (GRATUIT)
Inscription Gratuite se connecter
Merci à
Hiphigenie
/
malou Webmaster
pour avoir contribué à l'élaboration de cette fiche


Vous devez être membre accéder à ce service...

Pas encore inscrit ?

1 compte par personne, multi-compte interdit !

Ou identifiez-vous :


Rester sur la page

Inscription gratuite

Fiches en rapport

parmi 1674 fiches de maths

Désolé, votre version d'Internet Explorer est plus que périmée ! Merci de le mettre à jour ou de télécharger Firefox ou Google Chrome pour utiliser le site. Votre ordinateur vous remerciera !